12
$\begingroup$

We believe the answer to the following question, that is relevant to a joint research project with Piotr Szewczak, should be known. We would appreciate any help or pointer. Needed definitions may be found in, e.g., Blass's chapter in the Handbook of Set Theory.

For $f\in\omega^\omega$, let $K_f:=\{g\in \omega^\omega : g\le^* f\}$.

Let $\mathfrak{tmp}$ be the minimal cardinality of a set $Y\subseteq \omega^\omega$ such that the set $\bigcup_{f\in Y}K_f$ is not meager.

Since every set $K_f$ is meager, we have that $\mathfrak{b}\le \mathfrak{tmp}\le \mathrm{non}(meager)$.

Is $\mathfrak{tmp}$ (provably) equal to either of these cardinals?

$\endgroup$
5
  • 1
    $\begingroup$ You really aim at this to be a temporary name, eh? :-) $\endgroup$
    – Asaf Karagila
    Mar 26, 2015 at 23:22
  • $\begingroup$ Yes, we do. Such a clean definition - the cardinal must be known under some other (and better :) ) name. $\endgroup$ Mar 26, 2015 at 23:24
  • 1
    $\begingroup$ Note also that $\mathfrak{tmp} \leq \mathfrak{d}$. $\endgroup$ Mar 26, 2015 at 23:40
  • 2
    $\begingroup$ So $\mathfrak{tmp}<\mathrm{non}(meager)\,$ is consistent. $\endgroup$ Mar 26, 2015 at 23:51
  • $\begingroup$ @RamirodelaVega: Excellent point. In our application we work below $\mathfrak{d}$, so we missed this point. $\endgroup$ Mar 27, 2015 at 1:08

1 Answer 1

7
$\begingroup$

For every meager $M \subseteq \omega^{\omega}$ there exists $f_M \in \omega^{\omega}$ such that for every increasing $f \in \omega^{\omega}$, $K_f \subseteq M$ implies $f \leq^{\star} f_M$. For a proof, see Theorem 2.2.2 in Bartozynski, Judah book. It follows that your invariant is the bounding number.

$\endgroup$
1
  • $\begingroup$ Nice! Lyubomyr Zdomskyy points out another proof: $\omega^\omega$ is homeomorphic to $[\omega]^\omega$, and here $\mathfrak{b}$ is enough: Take a non-meager filter of character $\mathfrak{b}$ and note that each upper cone of a subset of $\omega$ is compact. $\endgroup$ Mar 27, 2015 at 14:51

Your Answer

By clicking “Post Your Answer”, you agree to our terms of service and acknowledge you have read our privacy policy.

Not the answer you're looking for? Browse other questions tagged or ask your own question.